Evaluate $\int_0^1\frac{\ln x\ln^2(1-x)}{1+x}dx$ in an elegant way

definite integralsintegrationpolylogarithmriemann-zetasequences-and-series

How to prove, in an elegant way that

$$I=\int_0^1\frac{\ln x\ln^2(1-x)}{1+x}dx=\frac{11}{4}\zeta(4)-\frac14\ln^42-6\operatorname{Li}_4\left(\frac12\right)\ ?$$


First, let me show you how I did it

\begin{align}
I&=\int_0^1\frac{\ln x\ln^2(1-x)}{1+x}\ dx\overset{1-x\ \mapsto x}{=}\int_0^1\frac{\ln(1-x)\ln^2x}{2-x}\ dx\\
&=\sum_{n=1}^\infty\frac1{2^n}\int_0^1x^{n-1}\ln^2x\ln(1-x)\ dx\\
&=\sum_{n=1}^\infty\frac1{2^n}\frac{\partial^2}{\partial n^2}\int_0^1x^{n-1}\ln(1-x)\ dx\\
&=\sum_{n=1}^\infty\frac1{2^n}\frac{\partial^2}{\partial n^2}\left(-\frac{H_n}{n}\right)\\
&=\sum_{n=1}^\infty\frac1{2^n}\left(\frac{2\zeta(2)}{n^2}+\frac{2\zeta(3)}{n}-\frac{2H_n}{n^32^n}-\frac{2H_n^{(2)}}{n^22^n}-\frac{2H_n^{(3)}}{n2^n}\right)\\
&=2\zeta(2)\operatorname{Li}_2\left(\frac12\right)+2\ln2\zeta(3)-2\sum_{n=1}^\infty\frac{H_n}{n^32^n}-2\sum_{n=1}^\infty\frac{H_n^{(2)}}{n^22^n}-2\sum_{n=1}^\infty\frac{H_n^{(3)}}{n2^n}
\end{align}

By substituting

$$S_1=\sum_{n=1}^\infty \frac{H_n}{n^32^n}=\operatorname{Li}_4\left(\frac12\right)+\frac18\zeta(4)-\frac18\ln2\zeta(3)+\frac1{24}\ln^42$$

$$
S_2=\sum_{n=1}^{\infty}\frac{H_n^{(2)}}{{n^22^n}}=\operatorname{Li_4}\left(\frac12\right)+\frac1{16}\zeta(4)+\frac14\ln2\zeta(3)-\frac14\ln^22\zeta(2)+\frac1{24}\ln^42$$

$$S_3=\sum_{n=1}^\infty\frac{H_n^{(3)}}{n2^n}=\operatorname{Li_4}\left(\frac12\right)-\frac{5}{16}\zeta(4)+\frac78\ln2\zeta(3)-\frac14\ln^22\zeta(2)+\frac{1}{24}\ln^42$$

along with $\operatorname{Li}_2(1/2)=\frac12\zeta(2)-\frac12\ln^22$ we get the closed form on $I$. Note that $S_1$, $S_2$ and $S_3$ can be found here, here and here respectively.


Now we can see how boring and tedious our calculations are as we used results of three harmonic series with powers of 2 in the denominator. A friend ( who proposed this problem ) suggested that the integral can be done without using harmonic series, so any idea how to do it that way?

Thanks

Best Answer

I'll just show an idea that avoids those type of sum, but skip the calculations. You might also have better ideas to solve them.

For start we will denote $a=\ln(1-x)$ and $b=\ln(1+x)$ and use the following identity: $$a^2=\frac12 (a+b)^2+\frac12(a-b)^2-b^2$$ $$\Rightarrow I=\frac12 \underbrace{\int_0^1 \frac{\ln x\ln^2(1-x^2)}{1+x}dx}_{I_1}+\frac12\underbrace{ \int_0^1 \frac{\ln x\ln^2\left(\frac{1-x}{1+x}\right)}{1+x}dx}_{I_2}-\underbrace{\int_0^1 \frac{\ln x\ln^2(1+x)}{1+x}dx}_{I_3}$$


For the first integral we will write the denominator as: $$\frac{1}{1+x}=\frac{1}{1-x^2}-\frac{x}{1-x^2}$$ $$\Rightarrow I_1=\int_0^1 \frac{\ln x\ln^2(1-x^2)}{1-x^2}dx-{\int_0^1 \frac{x\ln x\ln^2(1-x^2)}{1-x^2}dx}$$ $$\overset{x^2\to x}=\frac14 \int_0^1 \frac{\ln x\ln^2(1-x)}{1-x}\frac{dx}{\sqrt x}-\frac14\int_0^1 \frac{\ln x\ln^2(1-x)}{1-x}dx$$ Those two integral can be found using the second identity from here.


Let's also take $I_2$ and substitute $\frac{1-x}{1+x}\to x$. $$\Rightarrow I_2=\underbrace{\int_0^1 \frac{\ln(1-x)\ln^2 x}{1+x}dx}_{P}-\underbrace{\int_0^1 \frac{\ln(1+x)\ln^2 x}{1+x}dx}_{Q}$$ $$P-Q=I_2;\quad P+Q=\int_0^1 \frac{\ln(1-x^2)\ln^2 x}{1+x}dx$$ And again with the same trick done for $I_1$, we have: $$P+Q=\int_0^1 \frac{\ln(1-x^2)\ln^2 x}{1-x^2}dx-\int_0^1 \frac{x\ln(1-x^2)\ln^2 x}{1-x^2}dx$$ $$\overset{x^2\to x}=\frac18\int_0^1 \frac{\ln(1-x)\ln^2 x}{1-x}\frac{dx}{\sqrt x}-\frac18 \int_0^1 \frac{\ln(1-x)\ln^2 x}{1-x}dx$$ Henceforth we can extract our second integral, $I_2$ as: $$I_2=P-Q=(P+Q)-2Q$$ Note that $P+Q$ can again be found using the second identity from here.
Finally, we only need to find $Q$.

$$Q=\int_0^1 \frac{\ln(1+x)\ln^2 x}{1+x}dx=\sum_{n=1}^\infty (-1)^{n+1} H_n\int_0^1 x^{n}\ln^2 x=2\sum_{n=1}^\infty \frac{(-1)^{n+1}H_n}{(n+1)^3}$$ So $Q$ is actually an Euler sum in disguise, but you nicely found it here.


Also, $I_3$ is pretty easy, one just needs to use the same approach as done for $I_1$ in your following post. $$I_3=\int_0^1 \frac{\ln x \ln^2(1+x)}{1+x}dx\overset{IBP}=-\frac12\int_0^1 \frac{\ln^3(1+x)}{x}dx$$

Related Question